K
Khách

Hãy nhập câu hỏi của bạn vào đây, nếu là tài khoản VIP, bạn sẽ được ưu tiên trả lời.

AH
Akai Haruma
Giáo viên
4 tháng 1 2021

Bạn tham khảo lời giải tại đây:

cho \(x,y,z\ge0\) thỏa mãn \(x y z=6\). tìm GTLN và GTNN của biểu thức \(A=x^2 y^2 z^2\) - Hoc24

NV
17 tháng 12 2020

Với mọi x;y;z ta luôn có:

\(\left(x+y-1\right)^2+\left(z-\dfrac{1}{2}\right)^2\ge0\)

\(\Leftrightarrow x^2+y^2+2xy-2x-2y+1+z^2-z+\dfrac{1}{4}\ge0\)

\(\Leftrightarrow x^2+y^2+z^2+\dfrac{5}{4}+2xy-2x-2y-z\ge0\)

\(\Leftrightarrow2+2xy-2x-2y\ge z\)

\(\Leftrightarrow2\left(1-x\right)\left(1-y\right)\ge z\)

Dấu "=" xảy ra khi và chỉ khi \(x=y=z=\dfrac{1}{2}\)

 

24 tháng 12 2020

nhờ mn giúp mk bài này vs ạ

mk đang cần gấp !

cảm ơn mn nhiều

NV
25 tháng 12 2020

Đặt \(\left(\sqrt[3]{x};\sqrt[3]{y};\sqrt[3]{z}\right)=\left(a;b;c\right)\) \(\Rightarrow a^6+b^6+c^6=3\)

\(a^6+a^6+a^6+a^6+a^6+1\ge6a^5\)

Tương tự: \(5b^6+1\ge6b^5\) ; \(5c^6+1\ge6c^5\)

Cộng vế với vế: \(18=5\left(a^6+b^6+c^6\right)+3\ge6\left(a^5+b^5+c^5\right)\)

\(\Rightarrow3\ge a^5+b^6+b^5\)

BĐT cần chứng minh: \(\dfrac{a^3}{bc}+\dfrac{b^3}{ca}+\dfrac{c^3}{ab}\ge a^3b^3+b^3c^3+c^3a^3\) 

Ta có:

\(\dfrac{a^3}{bc}+\dfrac{b^3}{ca}+\dfrac{c^3}{ab}\ge\dfrac{ab}{c}+\dfrac{bc}{a}+\dfrac{ca}{b}\ge a+b+c\) (1)

Mà \(3\left(a+b+c\right)\ge\left(a^5+b^5+c^5\right)\left(a+b+c\right)\ge\left(a^3+b^3+c^3\right)^2\ge3\left(a^3b^3+b^3c^3+c^3a^3\right)\)

\(\Rightarrow a+b+c\ge a^3b^3+b^3c^3+c^3a^3\) (2)

Từ (1);(2) \(\Rightarrow\) đpcm

AH
Akai Haruma
Giáo viên
25 tháng 1 2021

Lời giải:Vì $x^2+y^2+z^2=2$ nên:

$P=\frac{x^2+y^2+z^2}{x^2+y^2}+\frac{x^2+y^2+z^2}{y^2+z^2}+\frac{x^2+y^2+z^2}{z^2+x^2}-\frac{x^3+y^3+z^3}{2xyz}$

$=3+\frac{x^2}{y^2+z^2}+\frac{y^2}{x^2+z^2}+\frac{z^2}{x^2+y^2}-\frac{x^3+y^3+z^3}{2xyz}$

$\leq 3+\frac{x^2}{2yz}+\frac{y^2}{2xz}+\frac{z^2}{2xy}-\frac{x^3+y^3+z^3}{2xyz}$

(theo BĐT AM-GM)

$=3+\frac{x^3+y^3+z^3}{2xyz}-\frac{x^3+y^3+z^3}{2xyz}=3$

Vậy $P_{\max}=3$

Dấu "=" xảy ra khi $x=y=z=\sqrt{\frac{2}{3}}$

 

14 tháng 2 2017

Ta có:

  P = 1 x ( 1 z 2 + 1 y 2 ) + 1 y ( 1 z 2 + 1 x 2 ) + 1 z ( 1 x 2 + 1 y 2 )

Đặt:  1 x = a ; 1 y = b ; 1 z = c  thì a,b,c>0 và a2+b2+c2=1

P = a b 2 + c 2 + b c 2 + a 2 + c a 2 + b 2 = a 2 a ( 1 − a 2 ) + b 2 b ( 1 − b 2 ) + c 2 c ( 1 − c 2 )

Áp dng bất đng thức Côsi cho 3 số dương ta có:

a 2 1 - a 2 2 = 1 2 .2 a 2 ( 1 − a 2 ) ( 1 − a 2 ) ≤ 1 2 2 a 2 + 1 − a 2 + 1 − a 2 3 = 4 27 = > a ( 1 − a 2 ) ≤ 2 3 3 < = > a 2 a ( 1 − a 2 ) ≥ 3 3 2 a 2 ( 1 )

Tương tự:  b 2 b ( 1 − b 2 ) ≥ 3 3 2 b 2 ( 2 ) ; c 2 c ( 1 − c 2 ) ≥ 3 3 2 c 2 ( 3 )

T (1); (2); (3) ta có  P ≥ 3 3 2 ( a 2 + b 2 + c 2 ) = 3 3 2

Đng thức xảy ra  a = b = c = 1 3 h a y   x = y = z = 3

Vậy giá trị nhỏ nhất của P là  3 3 2

AH
Akai Haruma
Giáo viên
29 tháng 5 2023

Đề lỗi công thức rồi. Bạn xem lại.

AH
Akai Haruma
Giáo viên
26 tháng 1 2021

Bạn tham khảo lời giải tại đây:

cho các số thực dưong x,y,z thỏa mãn : x2 y2 z2=3chứng minh rằng : \(\dfrac{x}{\sqrt[3]{yz}} \dfrac{y}{\sqrt[3]{zx}} \df... - Hoc24

AH
Akai Haruma
Giáo viên
26 tháng 1 2021

Cách khác:

Áp dụng BĐT AM-GM và BĐT Cauchy-Schwarz:

\(\sum \frac{x}{\sqrt[3]{yz}}\geq \sum \frac{x}{\frac{y+z+1}{3}}=3\sum \frac{x}{y+z+1}=3\sum \frac{x^2}{xy+xz+x}\)

\(\geq 3. \frac{(x+y+z)^2}{2(xy+yz+xz)+(x+y+z)}\)

Ta sẽ chứng minh: \(\frac{3(x+y+z)^2}{2(xy+yz+xz)+(x+y+z)}\geq xy+yz+xz(*)\)

Đặt $x+y+z=a$ thì $xy+yz+xz=\frac{a^2-3}{2}$

Bằng BĐT AM-GM dễ thấy $\sqrt{3}< a\leq 3$

BĐT $(*)$ trở thành:

$\frac{3a^2}{a^2+a-3}\geq \frac{a^2-3}{2}$

$\Leftrightarrow a^4+a^3-12a^2-3a+9\leq 0$

$\Leftrightarrow (a-3)(a+1)(a^2+3a-3)\leq 0$

Điều này đúng với mọi $\sqrt{3}< a\leq 3$

Do đó BĐT $(*)$ đúng nên ta có đpcm.

Dấu "=" xảy ra khi $x=y=z=1$